Allowed Publications
Slot System
Featured Buckets
Featured Buckets Admin

Getting a Leg up on the Diagnosis

Article Type
Changed
Wed, 12/04/2019 - 15:01
Display Headline
Getting a Leg up on the Diagnosis

Three years ago, lesions began appearing on this now 68-year-old woman’s legs. They have grown in size and number, and their roughness disturbs the patient. She has been told the lesions are related to aging, but she has never seen anything like them on her friends or family—and she is worried about what they might mean for her health.

Her primary care provider diagnosed warts and performed cryotherapy on several of the lesions. However, the pain was intolerable and the treatment ineffective. To add insult to injury, each treated spot blistered and took more than a month to heal, leaving behind a pinkish brown blemish.

In all other respects, the patient’s health is excellent.

EXAMINATION
Both legs, from the upper thighs to the tops of the feet, are covered with thousands of uniformly distributed, tiny, keratotic, rough, dry papules. All the lesions are essentially identical: white, with no associated signs of inflammation. The patient’s skin is quite dry in general. Neither her palms nor soles are affected.

Rough lesions on leg

What’s the diagnosis?

 

 

DISCUSSION
The most common problem seen in dermatology offices worldwide is seborrheic keratosis (SK), a totally benign epidermal excrescence that appears to be related to aging and heredity. Most patients are in their 50s when they first notice an SK, and with a bit of luck, they will only see a few in their lifetime. But some patients develop hundreds of SKs, many of which become quite large (3-5 cm) and unsightly. In certain circumstances, SKs can herald the arrival of an occult carcinoma (the Leser-Trelat sign).

This patient has what some consider a variant of SK, called stucco keratosis. These lesions manifest almost exclusively on the lower legs and feet—perhaps due to the relative lack of sebaceous glands in those areas—and most often on men older than 60. Distressing as they are, stucco keratoses have no pathologic implications.

Grossly and histologically, stucco keratoses are different from ordinary SKs. Each stucco keratosis lesion is essentially identical to the others, with a spiculated surface, white color, and average diameter of 2 to 3 mm. Histologically, they demonstrate a thickened epidermis with focal exophytic upward projections that resemble church spires. The lesions do not extend into the dermis.

Treatment of stucco keratoses is, at best, tedious, painful, and futile. The modalities used are cryotherapy or electrodessication with curettage. For a degree of comfort, use of a loofah after bathing will remove or smooth down a few lesions, but this process must be followed by application of a heavy emollient. Alas, regrowth is a certainty.

TAKE-HOME LEARNING POINTS

  • Stucco keratosis is considered a variant of seborrheic keratosis, although they differ in several significant ways.
  • The lesions of stucco keratosis are fairly uniform in appearance: white, rough, spiculated, epidermal papules measuring 2 to 3 mm.
  • Stucco keratoses affect about 10% of the population (men more often than women) and have no racial predilection or pathologic implications.
  • The lesions are found almost exclusively on the legs, from the knees down to and including the dorsa of the feet.
  • Treatment is far from satisfactory, for multiple reasons, including resultant pain and scarring.
Publications
Topics
Sections

Three years ago, lesions began appearing on this now 68-year-old woman’s legs. They have grown in size and number, and their roughness disturbs the patient. She has been told the lesions are related to aging, but she has never seen anything like them on her friends or family—and she is worried about what they might mean for her health.

Her primary care provider diagnosed warts and performed cryotherapy on several of the lesions. However, the pain was intolerable and the treatment ineffective. To add insult to injury, each treated spot blistered and took more than a month to heal, leaving behind a pinkish brown blemish.

In all other respects, the patient’s health is excellent.

EXAMINATION
Both legs, from the upper thighs to the tops of the feet, are covered with thousands of uniformly distributed, tiny, keratotic, rough, dry papules. All the lesions are essentially identical: white, with no associated signs of inflammation. The patient’s skin is quite dry in general. Neither her palms nor soles are affected.

Rough lesions on leg

What’s the diagnosis?

 

 

DISCUSSION
The most common problem seen in dermatology offices worldwide is seborrheic keratosis (SK), a totally benign epidermal excrescence that appears to be related to aging and heredity. Most patients are in their 50s when they first notice an SK, and with a bit of luck, they will only see a few in their lifetime. But some patients develop hundreds of SKs, many of which become quite large (3-5 cm) and unsightly. In certain circumstances, SKs can herald the arrival of an occult carcinoma (the Leser-Trelat sign).

This patient has what some consider a variant of SK, called stucco keratosis. These lesions manifest almost exclusively on the lower legs and feet—perhaps due to the relative lack of sebaceous glands in those areas—and most often on men older than 60. Distressing as they are, stucco keratoses have no pathologic implications.

Grossly and histologically, stucco keratoses are different from ordinary SKs. Each stucco keratosis lesion is essentially identical to the others, with a spiculated surface, white color, and average diameter of 2 to 3 mm. Histologically, they demonstrate a thickened epidermis with focal exophytic upward projections that resemble church spires. The lesions do not extend into the dermis.

Treatment of stucco keratoses is, at best, tedious, painful, and futile. The modalities used are cryotherapy or electrodessication with curettage. For a degree of comfort, use of a loofah after bathing will remove or smooth down a few lesions, but this process must be followed by application of a heavy emollient. Alas, regrowth is a certainty.

TAKE-HOME LEARNING POINTS

  • Stucco keratosis is considered a variant of seborrheic keratosis, although they differ in several significant ways.
  • The lesions of stucco keratosis are fairly uniform in appearance: white, rough, spiculated, epidermal papules measuring 2 to 3 mm.
  • Stucco keratoses affect about 10% of the population (men more often than women) and have no racial predilection or pathologic implications.
  • The lesions are found almost exclusively on the legs, from the knees down to and including the dorsa of the feet.
  • Treatment is far from satisfactory, for multiple reasons, including resultant pain and scarring.

Three years ago, lesions began appearing on this now 68-year-old woman’s legs. They have grown in size and number, and their roughness disturbs the patient. She has been told the lesions are related to aging, but she has never seen anything like them on her friends or family—and she is worried about what they might mean for her health.

Her primary care provider diagnosed warts and performed cryotherapy on several of the lesions. However, the pain was intolerable and the treatment ineffective. To add insult to injury, each treated spot blistered and took more than a month to heal, leaving behind a pinkish brown blemish.

In all other respects, the patient’s health is excellent.

EXAMINATION
Both legs, from the upper thighs to the tops of the feet, are covered with thousands of uniformly distributed, tiny, keratotic, rough, dry papules. All the lesions are essentially identical: white, with no associated signs of inflammation. The patient’s skin is quite dry in general. Neither her palms nor soles are affected.

Rough lesions on leg

What’s the diagnosis?

 

 

DISCUSSION
The most common problem seen in dermatology offices worldwide is seborrheic keratosis (SK), a totally benign epidermal excrescence that appears to be related to aging and heredity. Most patients are in their 50s when they first notice an SK, and with a bit of luck, they will only see a few in their lifetime. But some patients develop hundreds of SKs, many of which become quite large (3-5 cm) and unsightly. In certain circumstances, SKs can herald the arrival of an occult carcinoma (the Leser-Trelat sign).

This patient has what some consider a variant of SK, called stucco keratosis. These lesions manifest almost exclusively on the lower legs and feet—perhaps due to the relative lack of sebaceous glands in those areas—and most often on men older than 60. Distressing as they are, stucco keratoses have no pathologic implications.

Grossly and histologically, stucco keratoses are different from ordinary SKs. Each stucco keratosis lesion is essentially identical to the others, with a spiculated surface, white color, and average diameter of 2 to 3 mm. Histologically, they demonstrate a thickened epidermis with focal exophytic upward projections that resemble church spires. The lesions do not extend into the dermis.

Treatment of stucco keratoses is, at best, tedious, painful, and futile. The modalities used are cryotherapy or electrodessication with curettage. For a degree of comfort, use of a loofah after bathing will remove or smooth down a few lesions, but this process must be followed by application of a heavy emollient. Alas, regrowth is a certainty.

TAKE-HOME LEARNING POINTS

  • Stucco keratosis is considered a variant of seborrheic keratosis, although they differ in several significant ways.
  • The lesions of stucco keratosis are fairly uniform in appearance: white, rough, spiculated, epidermal papules measuring 2 to 3 mm.
  • Stucco keratoses affect about 10% of the population (men more often than women) and have no racial predilection or pathologic implications.
  • The lesions are found almost exclusively on the legs, from the knees down to and including the dorsa of the feet.
  • Treatment is far from satisfactory, for multiple reasons, including resultant pain and scarring.
Publications
Publications
Topics
Article Type
Display Headline
Getting a Leg up on the Diagnosis
Display Headline
Getting a Leg up on the Diagnosis
Sections
Disallow All Ads
Content Gating
No Gating (article Unlocked/Free)
Alternative CME
Disqus Comments
Default
Gate On Date
Wed, 12/04/2019 - 09:45
Un-Gate On Date
Wed, 12/04/2019 - 09:45
Use ProPublica
CFC Schedule Remove Status
Wed, 12/04/2019 - 09:45
Hide sidebar & use full width
render the right sidebar.

Keeping Lesions at Arm’s Length

Article Type
Changed
Thu, 11/21/2019 - 00:01
Display Headline
Keeping Lesions at Arm’s Length

A 14-year-old boy presents to dermatology for evaluation of an asymptomatic “rash” present on his arms since age 6. The condition has caught the attention of family members and teachers over the years, particularly in regard to possible contagion.

The patient is otherwise reasonably healthy, although he has asthma and seasonal allergies.

"Rash" on arm

EXAMINATION
The "rash" consists of uniformly distributed and sized planar papules. Although they are tiny, averaging only 1 mm wide, they are prominent enough to be noticeable and palpable. They appear slightly lighter than the surrounding skin. Distribution is from the lower deltoid to mid-dorsal forearm, affecting both arms identically. The volar aspects and triceps of both arms are totally spared.

The patient has type IV skin.

What’s the diagnosis?

 

 

DISCUSSION
This case is an almost perfect representation of lichen nitidus (LN), in terms of morphology, distribution, and configuration. Close examination of individual lesions revealed that the papules were somewhat planar (ie, flat-topped), giving their surfaces a reflective appearance that the eye interprets as white (particularly contrasted with darker skin).

LN can occur in anyone, but it is most often seen in those with darker skin. It is also frequently seen in children, many of whom are atopic, with dry, sensitive skin that is prone to eczema.

In terms of distribution, LN typically affects the extensor triceps, elbow, and forearms bilaterally. With its flat-topped and shiny appearance, LN is sometimes called "mini-lichen planus"—a condition that can demonstrate similar features. Fortunately, LN is seldom itchy and shares none of the distinct histologic characteristics of lichen planus.

LN is quite unusual, if not rare. It is also idiopathic and nearly always resolves on its own—although this can take months to years.

Emollients help to make the affected skin smoother and less visible. Class 4 steroid creams (eg, triamcinolone 0.05%) can help with itching.

TAKE-HOME LEARNING POINTS

  • Lichen nitidus (LN) is a rare idiopathic skin condition manifesting with patches of tiny planar papules; it typically affects the elbow and dorsal forearm.
  • LN has no pathologic implications and is asymptomatic and self-limited.
  • The lesions of LN have a “lichenoid” appearance—ie, a shiny, flat-topped look similar to that seen with lichen planus.
  • Fortunately, LN rarely requires treatment, aside from relief of mild itching.
Publications
Topics
Sections

A 14-year-old boy presents to dermatology for evaluation of an asymptomatic “rash” present on his arms since age 6. The condition has caught the attention of family members and teachers over the years, particularly in regard to possible contagion.

The patient is otherwise reasonably healthy, although he has asthma and seasonal allergies.

"Rash" on arm

EXAMINATION
The "rash" consists of uniformly distributed and sized planar papules. Although they are tiny, averaging only 1 mm wide, they are prominent enough to be noticeable and palpable. They appear slightly lighter than the surrounding skin. Distribution is from the lower deltoid to mid-dorsal forearm, affecting both arms identically. The volar aspects and triceps of both arms are totally spared.

The patient has type IV skin.

What’s the diagnosis?

 

 

DISCUSSION
This case is an almost perfect representation of lichen nitidus (LN), in terms of morphology, distribution, and configuration. Close examination of individual lesions revealed that the papules were somewhat planar (ie, flat-topped), giving their surfaces a reflective appearance that the eye interprets as white (particularly contrasted with darker skin).

LN can occur in anyone, but it is most often seen in those with darker skin. It is also frequently seen in children, many of whom are atopic, with dry, sensitive skin that is prone to eczema.

In terms of distribution, LN typically affects the extensor triceps, elbow, and forearms bilaterally. With its flat-topped and shiny appearance, LN is sometimes called "mini-lichen planus"—a condition that can demonstrate similar features. Fortunately, LN is seldom itchy and shares none of the distinct histologic characteristics of lichen planus.

LN is quite unusual, if not rare. It is also idiopathic and nearly always resolves on its own—although this can take months to years.

Emollients help to make the affected skin smoother and less visible. Class 4 steroid creams (eg, triamcinolone 0.05%) can help with itching.

TAKE-HOME LEARNING POINTS

  • Lichen nitidus (LN) is a rare idiopathic skin condition manifesting with patches of tiny planar papules; it typically affects the elbow and dorsal forearm.
  • LN has no pathologic implications and is asymptomatic and self-limited.
  • The lesions of LN have a “lichenoid” appearance—ie, a shiny, flat-topped look similar to that seen with lichen planus.
  • Fortunately, LN rarely requires treatment, aside from relief of mild itching.

A 14-year-old boy presents to dermatology for evaluation of an asymptomatic “rash” present on his arms since age 6. The condition has caught the attention of family members and teachers over the years, particularly in regard to possible contagion.

The patient is otherwise reasonably healthy, although he has asthma and seasonal allergies.

"Rash" on arm

EXAMINATION
The "rash" consists of uniformly distributed and sized planar papules. Although they are tiny, averaging only 1 mm wide, they are prominent enough to be noticeable and palpable. They appear slightly lighter than the surrounding skin. Distribution is from the lower deltoid to mid-dorsal forearm, affecting both arms identically. The volar aspects and triceps of both arms are totally spared.

The patient has type IV skin.

What’s the diagnosis?

 

 

DISCUSSION
This case is an almost perfect representation of lichen nitidus (LN), in terms of morphology, distribution, and configuration. Close examination of individual lesions revealed that the papules were somewhat planar (ie, flat-topped), giving their surfaces a reflective appearance that the eye interprets as white (particularly contrasted with darker skin).

LN can occur in anyone, but it is most often seen in those with darker skin. It is also frequently seen in children, many of whom are atopic, with dry, sensitive skin that is prone to eczema.

In terms of distribution, LN typically affects the extensor triceps, elbow, and forearms bilaterally. With its flat-topped and shiny appearance, LN is sometimes called "mini-lichen planus"—a condition that can demonstrate similar features. Fortunately, LN is seldom itchy and shares none of the distinct histologic characteristics of lichen planus.

LN is quite unusual, if not rare. It is also idiopathic and nearly always resolves on its own—although this can take months to years.

Emollients help to make the affected skin smoother and less visible. Class 4 steroid creams (eg, triamcinolone 0.05%) can help with itching.

TAKE-HOME LEARNING POINTS

  • Lichen nitidus (LN) is a rare idiopathic skin condition manifesting with patches of tiny planar papules; it typically affects the elbow and dorsal forearm.
  • LN has no pathologic implications and is asymptomatic and self-limited.
  • The lesions of LN have a “lichenoid” appearance—ie, a shiny, flat-topped look similar to that seen with lichen planus.
  • Fortunately, LN rarely requires treatment, aside from relief of mild itching.
Publications
Publications
Topics
Article Type
Display Headline
Keeping Lesions at Arm’s Length
Display Headline
Keeping Lesions at Arm’s Length
Sections
Disallow All Ads
Content Gating
No Gating (article Unlocked/Free)
Alternative CME
Disqus Comments
Default
Gate On Date
Wed, 11/20/2019 - 14:15
Un-Gate On Date
Wed, 11/20/2019 - 14:15
Use ProPublica
CFC Schedule Remove Status
Wed, 11/20/2019 - 14:15
Hide sidebar & use full width
render the right sidebar.

The Dog Can Stay, but the Rash Must Go

Article Type
Changed
Tue, 11/19/2019 - 11:45
Display Headline
The Dog Can Stay, but the Rash Must Go

A 50-year-old man presents with a 1-year history of an itchy, bumpy rash on his chest. He denies any history of similar rash and says there have been no “extraordinary changes” in his life that could have triggered this manifestation. Despite consulting various primary care providers, he has been unable to acquire either a definitive diagnosis or effective treatment.

The patient works exclusively in a climate-controlled office. Although there were no changes to laundry detergent, body soap, deodorant, or other products that might have precipitated the rash’s manifestation, he tried alternate products to see what effect they might have. Nothing beneficial came from these experiments. Similarly, the family dogs were temporarily “banished” with no improvement to his condition.

From the outset, the rash and the associated itching have been confined to the patient’s chest. No one else in his family is similarly affected.

The patient is otherwise quite well. He takes no prescription medications and denies any recent foreign travel.

Itchy, bumpy rash on chest

EXAMINATION
The papulovesicular rash is strikingly uniform. The patient’s entire chest is covered with tiny vesicles, many with clear fluid inside. The lesions average 1.2 to 2 mm in width, and nearly all are quite palpable. Each lesion is slightly erythematous but neither warm nor tender on palpation.

Examination of the rest of the patient’s exposed skin reveals no similar lesions. His back, hands, and genitals are notably free of any such lesions.

A shave biopsy is performed, utilizing a saucerization technique, and the specimen is submitted to pathology for routine processing. The report confirms the papulovesicular nature of the lesions—but more significantly, it shows consistent acantholysis (loss of intracellular connections between keratinocytes), along with focal lymphohistiocytic infiltrates.

What’s the diagnosis?

 

 

DISCUSSION
This is a classic presentation of Grover disease, also known as transient acantholytic dermatosis (AD). While not rare, it is seen only occasionally in dermatology practices. When it does walk through the door, it is twice as likely to be seen in a male than in a female patient and less commonly seen in those with darker skin.

AD is easy enough to diagnose clinically, without biopsy, particularly in classic cases such as this one. The distribution and morphology of the rash, as well as the gender and age of the patient, are all typical of this idiopathic condition. The biopsy results, besides being consistent with AD, did serve to rule out other items in the differential (eg, bacterial folliculitis, pemphigus, and acne).

Since AD was first described in 1974 by R.W. Grover, MD, much research has been conducted to flesh out the nature of the disease, its potential causes, and possible treatment. One certainty about so-called transient AD is that most cases are far from transient—in fact, they can last for a year or more. Attempts have been made to connect AD with internal disease (eg, occult malignancy) or even mercury exposure, but these theories have not been corroborated.

Consistent treatment success has also been elusive. Most patients achieve decent relief with the use of topical steroid creams, with or without the addition of anti-inflammatory medications (eg, doxycycline). Other options include isotretinoin and psoralen plus ultraviolet A (PUVA) photochemotherapy. Fortunately, most cases eventually clear up.

TAKE-HOME LEARNING POINTS

  • Grover disease, also known as transient acantholytic dermatosis (AD), usually manifests with an acute eruption of papulovesicular lesions.
  • AD lesions tend to be confined to the chest and are typically pruritic.
  • Clinical diagnosis is usually adequate, although biopsy, which will reveal typical findings of acantholysis, may be necessary to rule out other items in the differential.
  • Treatment with topical steroids, oral doxycycline, and “tincture of time” usually suffices, but resolution may take a year or more.
Publications
Topics
Sections

A 50-year-old man presents with a 1-year history of an itchy, bumpy rash on his chest. He denies any history of similar rash and says there have been no “extraordinary changes” in his life that could have triggered this manifestation. Despite consulting various primary care providers, he has been unable to acquire either a definitive diagnosis or effective treatment.

The patient works exclusively in a climate-controlled office. Although there were no changes to laundry detergent, body soap, deodorant, or other products that might have precipitated the rash’s manifestation, he tried alternate products to see what effect they might have. Nothing beneficial came from these experiments. Similarly, the family dogs were temporarily “banished” with no improvement to his condition.

From the outset, the rash and the associated itching have been confined to the patient’s chest. No one else in his family is similarly affected.

The patient is otherwise quite well. He takes no prescription medications and denies any recent foreign travel.

Itchy, bumpy rash on chest

EXAMINATION
The papulovesicular rash is strikingly uniform. The patient’s entire chest is covered with tiny vesicles, many with clear fluid inside. The lesions average 1.2 to 2 mm in width, and nearly all are quite palpable. Each lesion is slightly erythematous but neither warm nor tender on palpation.

Examination of the rest of the patient’s exposed skin reveals no similar lesions. His back, hands, and genitals are notably free of any such lesions.

A shave biopsy is performed, utilizing a saucerization technique, and the specimen is submitted to pathology for routine processing. The report confirms the papulovesicular nature of the lesions—but more significantly, it shows consistent acantholysis (loss of intracellular connections between keratinocytes), along with focal lymphohistiocytic infiltrates.

What’s the diagnosis?

 

 

DISCUSSION
This is a classic presentation of Grover disease, also known as transient acantholytic dermatosis (AD). While not rare, it is seen only occasionally in dermatology practices. When it does walk through the door, it is twice as likely to be seen in a male than in a female patient and less commonly seen in those with darker skin.

AD is easy enough to diagnose clinically, without biopsy, particularly in classic cases such as this one. The distribution and morphology of the rash, as well as the gender and age of the patient, are all typical of this idiopathic condition. The biopsy results, besides being consistent with AD, did serve to rule out other items in the differential (eg, bacterial folliculitis, pemphigus, and acne).

Since AD was first described in 1974 by R.W. Grover, MD, much research has been conducted to flesh out the nature of the disease, its potential causes, and possible treatment. One certainty about so-called transient AD is that most cases are far from transient—in fact, they can last for a year or more. Attempts have been made to connect AD with internal disease (eg, occult malignancy) or even mercury exposure, but these theories have not been corroborated.

Consistent treatment success has also been elusive. Most patients achieve decent relief with the use of topical steroid creams, with or without the addition of anti-inflammatory medications (eg, doxycycline). Other options include isotretinoin and psoralen plus ultraviolet A (PUVA) photochemotherapy. Fortunately, most cases eventually clear up.

TAKE-HOME LEARNING POINTS

  • Grover disease, also known as transient acantholytic dermatosis (AD), usually manifests with an acute eruption of papulovesicular lesions.
  • AD lesions tend to be confined to the chest and are typically pruritic.
  • Clinical diagnosis is usually adequate, although biopsy, which will reveal typical findings of acantholysis, may be necessary to rule out other items in the differential.
  • Treatment with topical steroids, oral doxycycline, and “tincture of time” usually suffices, but resolution may take a year or more.

A 50-year-old man presents with a 1-year history of an itchy, bumpy rash on his chest. He denies any history of similar rash and says there have been no “extraordinary changes” in his life that could have triggered this manifestation. Despite consulting various primary care providers, he has been unable to acquire either a definitive diagnosis or effective treatment.

The patient works exclusively in a climate-controlled office. Although there were no changes to laundry detergent, body soap, deodorant, or other products that might have precipitated the rash’s manifestation, he tried alternate products to see what effect they might have. Nothing beneficial came from these experiments. Similarly, the family dogs were temporarily “banished” with no improvement to his condition.

From the outset, the rash and the associated itching have been confined to the patient’s chest. No one else in his family is similarly affected.

The patient is otherwise quite well. He takes no prescription medications and denies any recent foreign travel.

Itchy, bumpy rash on chest

EXAMINATION
The papulovesicular rash is strikingly uniform. The patient’s entire chest is covered with tiny vesicles, many with clear fluid inside. The lesions average 1.2 to 2 mm in width, and nearly all are quite palpable. Each lesion is slightly erythematous but neither warm nor tender on palpation.

Examination of the rest of the patient’s exposed skin reveals no similar lesions. His back, hands, and genitals are notably free of any such lesions.

A shave biopsy is performed, utilizing a saucerization technique, and the specimen is submitted to pathology for routine processing. The report confirms the papulovesicular nature of the lesions—but more significantly, it shows consistent acantholysis (loss of intracellular connections between keratinocytes), along with focal lymphohistiocytic infiltrates.

What’s the diagnosis?

 

 

DISCUSSION
This is a classic presentation of Grover disease, also known as transient acantholytic dermatosis (AD). While not rare, it is seen only occasionally in dermatology practices. When it does walk through the door, it is twice as likely to be seen in a male than in a female patient and less commonly seen in those with darker skin.

AD is easy enough to diagnose clinically, without biopsy, particularly in classic cases such as this one. The distribution and morphology of the rash, as well as the gender and age of the patient, are all typical of this idiopathic condition. The biopsy results, besides being consistent with AD, did serve to rule out other items in the differential (eg, bacterial folliculitis, pemphigus, and acne).

Since AD was first described in 1974 by R.W. Grover, MD, much research has been conducted to flesh out the nature of the disease, its potential causes, and possible treatment. One certainty about so-called transient AD is that most cases are far from transient—in fact, they can last for a year or more. Attempts have been made to connect AD with internal disease (eg, occult malignancy) or even mercury exposure, but these theories have not been corroborated.

Consistent treatment success has also been elusive. Most patients achieve decent relief with the use of topical steroid creams, with or without the addition of anti-inflammatory medications (eg, doxycycline). Other options include isotretinoin and psoralen plus ultraviolet A (PUVA) photochemotherapy. Fortunately, most cases eventually clear up.

TAKE-HOME LEARNING POINTS

  • Grover disease, also known as transient acantholytic dermatosis (AD), usually manifests with an acute eruption of papulovesicular lesions.
  • AD lesions tend to be confined to the chest and are typically pruritic.
  • Clinical diagnosis is usually adequate, although biopsy, which will reveal typical findings of acantholysis, may be necessary to rule out other items in the differential.
  • Treatment with topical steroids, oral doxycycline, and “tincture of time” usually suffices, but resolution may take a year or more.
Publications
Publications
Topics
Article Type
Display Headline
The Dog Can Stay, but the Rash Must Go
Display Headline
The Dog Can Stay, but the Rash Must Go
Sections
Disallow All Ads
Content Gating
No Gating (article Unlocked/Free)
Alternative CME
Disqus Comments
Default
Gate On Date
Thu, 11/07/2019 - 08:15
Un-Gate On Date
Thu, 11/07/2019 - 08:15
Use ProPublica
CFC Schedule Remove Status
Thu, 11/07/2019 - 08:15
Hide sidebar & use full width
render the right sidebar.

Boy, Is My Face Red!

Article Type
Changed
Thu, 10/24/2019 - 08:09
Display Headline
Boy, Is My Face Red!

A 17-year-old boy was born with rough skin on his face, arms, legs, thighs, and posterior shoulders. Over the years, his face, especially the posterior lateral aspects, has become progressively redder, while the roughness has increased. The redness is amplified with heat, exertion, anger, or embarrassment. Regarding the latter, mere mention of the condition by his siblings results in worsening of the erythema. Additionally, the skin in his eyebrows is now turning red and scaly.

The patient denies a history of dandruff. His parents, who have accompanied him to the clinic, report a family history of similar skin changes on triceps and thighs, but not on faces. There is no family history of cardiac anomalies or other congenital abnormalities. The boy’s health is otherwise excellent.

Red and scaly eyebrows on boy's face

EXAMINATION
The patient’s bilateral triceps are covered with fine, rough, follicular papules, which create a faintly erythematous look. Similar lesions are visible on his posterior shoulders and anterior thighs. The skin beneath his eyebrows is faintly erythematous and scaly.

The posterior sides of his face are bright red and covered with the same type of papules. The erythema grows redder as it approaches the immediate preauricular areas, where it ends abruptly, creating a sharp demarcation with the white skin closer to the ears. The visual effect is almost clownish, as if bright red makeup had been applied.

Rough skin on boy's face

What’s the diagnosis?

 

 

DISCUSSION
This young man has all the signs of an extremely rare variant of keratosis pilaris (KP) called ulerythema ophryogenes (UO). In the United States, about 40% of adults have ordinary KP, which usually manifests in childhood (with about 80% of cases occurring in adolescent girls). KP is inherited through autosomal dominance, with highly variable penetrance, though no specific gene has been identified. In this form, KP is considered by most experts to be a minor diagnostic criterion for atopic dermatitis.

However, UO is not merely a variant of KP. Over the decades, it has been connected with more serious conditions, such as cardiofaciocutaneous syndrome, Rubinstein-Taybi syndrome, and Cornelia de Lange disease. Although these conditions are not common, they should be considered when UO is seen.

For this patient, the main concern was his appearance, especially the pronounced erythema around the periphery of his face. This aspect of the problem was addressed with a referral to a provider who can, using an assortment of lasers, try to even out his skin color and hopefully erase the sharp border at the periphery of the affected area.

For the physical discomfort caused by UO, the patient was instructed either to use general moisturizers to reduce dryness or to consider using moisturizers containing salicylic acid, which should help to reduce the prominence of the papules. For the erythema in his brows, he is using 2.5% hydrocortisone ointment two to three times a week.

TAKE-HOME LEARNING POINTS

  • Ulerythema ophryogenes is a rarely encountered variant of keratosis pilaris—a condition inherited by autosomal dominance with highly variable penetrance.
  • Its main significance, beyond cosmetic concerns, is the possible connection with syndromes that involve heart and structural defects (eg, cardiofaciocutaneous syndrome).
  • Treatment options include heavy emollients to soften the scaly papules and laser therapy to reduce the extreme redness seen on the periphery of the face.
Publications
Topics
Sections

A 17-year-old boy was born with rough skin on his face, arms, legs, thighs, and posterior shoulders. Over the years, his face, especially the posterior lateral aspects, has become progressively redder, while the roughness has increased. The redness is amplified with heat, exertion, anger, or embarrassment. Regarding the latter, mere mention of the condition by his siblings results in worsening of the erythema. Additionally, the skin in his eyebrows is now turning red and scaly.

The patient denies a history of dandruff. His parents, who have accompanied him to the clinic, report a family history of similar skin changes on triceps and thighs, but not on faces. There is no family history of cardiac anomalies or other congenital abnormalities. The boy’s health is otherwise excellent.

Red and scaly eyebrows on boy's face

EXAMINATION
The patient’s bilateral triceps are covered with fine, rough, follicular papules, which create a faintly erythematous look. Similar lesions are visible on his posterior shoulders and anterior thighs. The skin beneath his eyebrows is faintly erythematous and scaly.

The posterior sides of his face are bright red and covered with the same type of papules. The erythema grows redder as it approaches the immediate preauricular areas, where it ends abruptly, creating a sharp demarcation with the white skin closer to the ears. The visual effect is almost clownish, as if bright red makeup had been applied.

Rough skin on boy's face

What’s the diagnosis?

 

 

DISCUSSION
This young man has all the signs of an extremely rare variant of keratosis pilaris (KP) called ulerythema ophryogenes (UO). In the United States, about 40% of adults have ordinary KP, which usually manifests in childhood (with about 80% of cases occurring in adolescent girls). KP is inherited through autosomal dominance, with highly variable penetrance, though no specific gene has been identified. In this form, KP is considered by most experts to be a minor diagnostic criterion for atopic dermatitis.

However, UO is not merely a variant of KP. Over the decades, it has been connected with more serious conditions, such as cardiofaciocutaneous syndrome, Rubinstein-Taybi syndrome, and Cornelia de Lange disease. Although these conditions are not common, they should be considered when UO is seen.

For this patient, the main concern was his appearance, especially the pronounced erythema around the periphery of his face. This aspect of the problem was addressed with a referral to a provider who can, using an assortment of lasers, try to even out his skin color and hopefully erase the sharp border at the periphery of the affected area.

For the physical discomfort caused by UO, the patient was instructed either to use general moisturizers to reduce dryness or to consider using moisturizers containing salicylic acid, which should help to reduce the prominence of the papules. For the erythema in his brows, he is using 2.5% hydrocortisone ointment two to three times a week.

TAKE-HOME LEARNING POINTS

  • Ulerythema ophryogenes is a rarely encountered variant of keratosis pilaris—a condition inherited by autosomal dominance with highly variable penetrance.
  • Its main significance, beyond cosmetic concerns, is the possible connection with syndromes that involve heart and structural defects (eg, cardiofaciocutaneous syndrome).
  • Treatment options include heavy emollients to soften the scaly papules and laser therapy to reduce the extreme redness seen on the periphery of the face.

A 17-year-old boy was born with rough skin on his face, arms, legs, thighs, and posterior shoulders. Over the years, his face, especially the posterior lateral aspects, has become progressively redder, while the roughness has increased. The redness is amplified with heat, exertion, anger, or embarrassment. Regarding the latter, mere mention of the condition by his siblings results in worsening of the erythema. Additionally, the skin in his eyebrows is now turning red and scaly.

The patient denies a history of dandruff. His parents, who have accompanied him to the clinic, report a family history of similar skin changes on triceps and thighs, but not on faces. There is no family history of cardiac anomalies or other congenital abnormalities. The boy’s health is otherwise excellent.

Red and scaly eyebrows on boy's face

EXAMINATION
The patient’s bilateral triceps are covered with fine, rough, follicular papules, which create a faintly erythematous look. Similar lesions are visible on his posterior shoulders and anterior thighs. The skin beneath his eyebrows is faintly erythematous and scaly.

The posterior sides of his face are bright red and covered with the same type of papules. The erythema grows redder as it approaches the immediate preauricular areas, where it ends abruptly, creating a sharp demarcation with the white skin closer to the ears. The visual effect is almost clownish, as if bright red makeup had been applied.

Rough skin on boy's face

What’s the diagnosis?

 

 

DISCUSSION
This young man has all the signs of an extremely rare variant of keratosis pilaris (KP) called ulerythema ophryogenes (UO). In the United States, about 40% of adults have ordinary KP, which usually manifests in childhood (with about 80% of cases occurring in adolescent girls). KP is inherited through autosomal dominance, with highly variable penetrance, though no specific gene has been identified. In this form, KP is considered by most experts to be a minor diagnostic criterion for atopic dermatitis.

However, UO is not merely a variant of KP. Over the decades, it has been connected with more serious conditions, such as cardiofaciocutaneous syndrome, Rubinstein-Taybi syndrome, and Cornelia de Lange disease. Although these conditions are not common, they should be considered when UO is seen.

For this patient, the main concern was his appearance, especially the pronounced erythema around the periphery of his face. This aspect of the problem was addressed with a referral to a provider who can, using an assortment of lasers, try to even out his skin color and hopefully erase the sharp border at the periphery of the affected area.

For the physical discomfort caused by UO, the patient was instructed either to use general moisturizers to reduce dryness or to consider using moisturizers containing salicylic acid, which should help to reduce the prominence of the papules. For the erythema in his brows, he is using 2.5% hydrocortisone ointment two to three times a week.

TAKE-HOME LEARNING POINTS

  • Ulerythema ophryogenes is a rarely encountered variant of keratosis pilaris—a condition inherited by autosomal dominance with highly variable penetrance.
  • Its main significance, beyond cosmetic concerns, is the possible connection with syndromes that involve heart and structural defects (eg, cardiofaciocutaneous syndrome).
  • Treatment options include heavy emollients to soften the scaly papules and laser therapy to reduce the extreme redness seen on the periphery of the face.
Publications
Publications
Topics
Article Type
Display Headline
Boy, Is My Face Red!
Display Headline
Boy, Is My Face Red!
Sections
Disallow All Ads
Content Gating
No Gating (article Unlocked/Free)
Alternative CME
Disqus Comments
Default
Gate On Date
Wed, 10/23/2019 - 15:45
Un-Gate On Date
Wed, 10/23/2019 - 15:45
Use ProPublica
CFC Schedule Remove Status
Wed, 10/23/2019 - 15:45
Hide sidebar & use full width
render the right sidebar.

Start From Scratch

Article Type
Changed
Thu, 10/10/2019 - 09:29
Display Headline
Start From Scratch

A 50-year-old man presents with complaints of a rash on his right leg that manifested 20 years ago. Although the rash is worrisome, he says the associated pruritus is worse. During the workday, he is able to ignore the itching—but the minute he gets home, he begins to scratch.

He knows the scratching is counterproductive in the long run, but the urge to do it is quite compelling. Sometimes he uses a wet washcloth; other times, he will actually use a hair brush to satiate the itching. The relief is intensely satisfying albeit short lived.

The rash has persisted despite multiple treatment attempts. Tried products include OTC moisturizers and antifungal creams, as well as prescription antifungal creams. None has had an effect.

The patient denies any other skin problems. He does recall having eczema as a child. Although that has long since resolved, he remains quite allergy prone and is particularly sensitive to airborne allergens—a trait that runs strongly in his family.

Rash on right leg

EXAMINATION
A pink, oval rash covers most of the patient’s right lateral calf. It has a thickened, faintly scaly surface that is uniform and sharply circumscribed. There is no increased warmth or tenderness on palpation. No lymph nodes can be felt in the right groin. A check of the patient’s knees, elbows, scalp, nails, and trunk show no sign of rash or other changes.

What’s the diagnosis?

 

 

DISCUSSION
Lichen simplex chronicus (LSC), previously known as neurodermatitis, is quite common but frequently misdiagnosed. Patients often report that their condition started with a bug bite or poison ivy—a provocation that gets the patient in the habit of scratching, which continues long after the initial outbreak has subsided. Thus, LSC is often associated with significant chronicity, as typified by this case.

What patients seldom understand is their own role in the perpetuation of their condition. The urge to scratch is so unbearable that few can resist it. Over time, the scratching creates more nerves that have a lower threshold for itching, and thus the itch-scratch-itch cycle is born. Many LSC patients are atopic, which predisposes them to itching in general and to xerosis especially.

The literature asserts that LSC affects the genders equally, but this ignores the fact that it can present significantly differently in men and women. This patient’s area of involvement is quite typical for men, most of whom never moisturize and for whom the lateral calf is readily accessible. In the author’s experience, the most common location for LSC in women is the nuchal scalp, where heat and sweat appear to play a role, along with ready accessibility to fingernails or the sharp end of a pencil. Other common areas of involvement include the dorsal forearms and the scrotum or vulvae.

Biopsy is seldom necessary, but if performed, it will show a marked thickening of the epidermis, orthokeratosis (normal keratinocytes about to shed), and compacted, elongated rete ridges. These and other changes effectively rule out other items in the differential (eg, psoriasis, simple eczema, fungal infection).

Stopping the itch-scratch-itch cycle with mid-strength topical steroid creams or foams is the first step in treating LSC. But then the patient must be convinced of his contribution to the treatment: leaving the affected sites alone. Truth be told, after 20 years of scratching, the best this patient can look forward to is some relief—not only from the itching, but also from concern about all the terrible things he now knows he doesn’t have.

TAKE-HOME LEARNING POINTS

  • Lichen simplex chronicus is quite common in both genders and is typified by longstanding severe itching, usually confined to one area.
  • Atopy, xerosis, and stress all appear to contribute to the problem.
  • Stopping the itch-scratch-itch cycle with topical steroids is a key component of treatment.
  • Patient education—on the nature of the problem and the patient’s role in controlling it—is just as important as any prescribed medication.
Publications
Topics
Sections

A 50-year-old man presents with complaints of a rash on his right leg that manifested 20 years ago. Although the rash is worrisome, he says the associated pruritus is worse. During the workday, he is able to ignore the itching—but the minute he gets home, he begins to scratch.

He knows the scratching is counterproductive in the long run, but the urge to do it is quite compelling. Sometimes he uses a wet washcloth; other times, he will actually use a hair brush to satiate the itching. The relief is intensely satisfying albeit short lived.

The rash has persisted despite multiple treatment attempts. Tried products include OTC moisturizers and antifungal creams, as well as prescription antifungal creams. None has had an effect.

The patient denies any other skin problems. He does recall having eczema as a child. Although that has long since resolved, he remains quite allergy prone and is particularly sensitive to airborne allergens—a trait that runs strongly in his family.

Rash on right leg

EXAMINATION
A pink, oval rash covers most of the patient’s right lateral calf. It has a thickened, faintly scaly surface that is uniform and sharply circumscribed. There is no increased warmth or tenderness on palpation. No lymph nodes can be felt in the right groin. A check of the patient’s knees, elbows, scalp, nails, and trunk show no sign of rash or other changes.

What’s the diagnosis?

 

 

DISCUSSION
Lichen simplex chronicus (LSC), previously known as neurodermatitis, is quite common but frequently misdiagnosed. Patients often report that their condition started with a bug bite or poison ivy—a provocation that gets the patient in the habit of scratching, which continues long after the initial outbreak has subsided. Thus, LSC is often associated with significant chronicity, as typified by this case.

What patients seldom understand is their own role in the perpetuation of their condition. The urge to scratch is so unbearable that few can resist it. Over time, the scratching creates more nerves that have a lower threshold for itching, and thus the itch-scratch-itch cycle is born. Many LSC patients are atopic, which predisposes them to itching in general and to xerosis especially.

The literature asserts that LSC affects the genders equally, but this ignores the fact that it can present significantly differently in men and women. This patient’s area of involvement is quite typical for men, most of whom never moisturize and for whom the lateral calf is readily accessible. In the author’s experience, the most common location for LSC in women is the nuchal scalp, where heat and sweat appear to play a role, along with ready accessibility to fingernails or the sharp end of a pencil. Other common areas of involvement include the dorsal forearms and the scrotum or vulvae.

Biopsy is seldom necessary, but if performed, it will show a marked thickening of the epidermis, orthokeratosis (normal keratinocytes about to shed), and compacted, elongated rete ridges. These and other changes effectively rule out other items in the differential (eg, psoriasis, simple eczema, fungal infection).

Stopping the itch-scratch-itch cycle with mid-strength topical steroid creams or foams is the first step in treating LSC. But then the patient must be convinced of his contribution to the treatment: leaving the affected sites alone. Truth be told, after 20 years of scratching, the best this patient can look forward to is some relief—not only from the itching, but also from concern about all the terrible things he now knows he doesn’t have.

TAKE-HOME LEARNING POINTS

  • Lichen simplex chronicus is quite common in both genders and is typified by longstanding severe itching, usually confined to one area.
  • Atopy, xerosis, and stress all appear to contribute to the problem.
  • Stopping the itch-scratch-itch cycle with topical steroids is a key component of treatment.
  • Patient education—on the nature of the problem and the patient’s role in controlling it—is just as important as any prescribed medication.

A 50-year-old man presents with complaints of a rash on his right leg that manifested 20 years ago. Although the rash is worrisome, he says the associated pruritus is worse. During the workday, he is able to ignore the itching—but the minute he gets home, he begins to scratch.

He knows the scratching is counterproductive in the long run, but the urge to do it is quite compelling. Sometimes he uses a wet washcloth; other times, he will actually use a hair brush to satiate the itching. The relief is intensely satisfying albeit short lived.

The rash has persisted despite multiple treatment attempts. Tried products include OTC moisturizers and antifungal creams, as well as prescription antifungal creams. None has had an effect.

The patient denies any other skin problems. He does recall having eczema as a child. Although that has long since resolved, he remains quite allergy prone and is particularly sensitive to airborne allergens—a trait that runs strongly in his family.

Rash on right leg

EXAMINATION
A pink, oval rash covers most of the patient’s right lateral calf. It has a thickened, faintly scaly surface that is uniform and sharply circumscribed. There is no increased warmth or tenderness on palpation. No lymph nodes can be felt in the right groin. A check of the patient’s knees, elbows, scalp, nails, and trunk show no sign of rash or other changes.

What’s the diagnosis?

 

 

DISCUSSION
Lichen simplex chronicus (LSC), previously known as neurodermatitis, is quite common but frequently misdiagnosed. Patients often report that their condition started with a bug bite or poison ivy—a provocation that gets the patient in the habit of scratching, which continues long after the initial outbreak has subsided. Thus, LSC is often associated with significant chronicity, as typified by this case.

What patients seldom understand is their own role in the perpetuation of their condition. The urge to scratch is so unbearable that few can resist it. Over time, the scratching creates more nerves that have a lower threshold for itching, and thus the itch-scratch-itch cycle is born. Many LSC patients are atopic, which predisposes them to itching in general and to xerosis especially.

The literature asserts that LSC affects the genders equally, but this ignores the fact that it can present significantly differently in men and women. This patient’s area of involvement is quite typical for men, most of whom never moisturize and for whom the lateral calf is readily accessible. In the author’s experience, the most common location for LSC in women is the nuchal scalp, where heat and sweat appear to play a role, along with ready accessibility to fingernails or the sharp end of a pencil. Other common areas of involvement include the dorsal forearms and the scrotum or vulvae.

Biopsy is seldom necessary, but if performed, it will show a marked thickening of the epidermis, orthokeratosis (normal keratinocytes about to shed), and compacted, elongated rete ridges. These and other changes effectively rule out other items in the differential (eg, psoriasis, simple eczema, fungal infection).

Stopping the itch-scratch-itch cycle with mid-strength topical steroid creams or foams is the first step in treating LSC. But then the patient must be convinced of his contribution to the treatment: leaving the affected sites alone. Truth be told, after 20 years of scratching, the best this patient can look forward to is some relief—not only from the itching, but also from concern about all the terrible things he now knows he doesn’t have.

TAKE-HOME LEARNING POINTS

  • Lichen simplex chronicus is quite common in both genders and is typified by longstanding severe itching, usually confined to one area.
  • Atopy, xerosis, and stress all appear to contribute to the problem.
  • Stopping the itch-scratch-itch cycle with topical steroids is a key component of treatment.
  • Patient education—on the nature of the problem and the patient’s role in controlling it—is just as important as any prescribed medication.
Publications
Publications
Topics
Article Type
Display Headline
Start From Scratch
Display Headline
Start From Scratch
Sections
Disallow All Ads
Content Gating
No Gating (article Unlocked/Free)
Alternative CME
Disqus Comments
Default
Gate On Date
Thu, 10/10/2019 - 09:15
Un-Gate On Date
Thu, 10/10/2019 - 09:15
Use ProPublica
CFC Schedule Remove Status
Thu, 10/10/2019 - 09:15
Hide sidebar & use full width
render the right sidebar.

Blips on Lips

Article Type
Changed
Thu, 09/26/2019 - 00:01
Display Headline
Blips on Lips

A 39-year-old woman presents with red vascular streaks on her upper and lower lips. They have slowly multiplied and become more prominent since she first noticed them several years ago. Although the lesions are asymptomatic, their effect on her appearance bothers the patient.

Her primary care provider tried to resolve the problem with cryotherapy. But the treatment attempt was unsuccessful.

During history-taking, the patient divulges other existing health problems. She has been diagnosed with Raynaud syndrome, which flares several times a year, especially in cold weather or times of exceptional stress. She also has permanent thickening of the skin on her distal fingers, a result of sudden-onset swelling of all 10 fingers several years ago.

She denies any problems with eating, such as heartburn or difficulty swallowing. She also denies any respiratory problems or chronic fatigue.

Red vascular streaks on upper and lower lips

EXAMINATION
Seven very slender telangiectasias, most aligned vertically, are seen on the upper and lower vermillion surfaces. They range from a pinpoint to 6 mm in length. There are no similar lesions are on the rest of the oral mucosae, the face, or the chest.

The patient’s fingers, from the metacarpals to the tips, are decidedly edematous and firm but not tender. Several fingertips are scarred from past Raynaud episodes.

Swollen and scarred fingertips from Raynaud episodes

The patient looks her stated age and is in no distress.

What’s the diagnosis?

 

 

DISCUSSION
This patient almost certainly has CREST syndrome, a limited form of systemic sclerosis (or scleroderma). Both represent an autoimmune process in which antibodies attack cell DNA and centromeres (a component of the cell nucleus).

CREST can be difficult to diagnose because it can involve diverse organ systems. The name of the syndrome is an acronym for the symptoms it causes:

Calcinosis is a deposition of calcium triggered by inflammation. It manifests as small subcutaneous nodules, which are usually felt on the hands or seen on radiographs of the hands.

Raynaud syndrome causes intense vasoconstriction of blood vessels, usually in fingertips or toes, which first turn white, then red. The phenomenon is accompanied by pain and can be triggered by cold or stress.

Esophageal dysmotility occurs when atrophy and/or fibrosis of the esophageal lining leads to difficulty swallowing food.

Sclerodactyly is the term for tightening and thickening of the skin on the hands and fingers.

Telangiectasias are dilated capillaries that can worsen with time; they are common on lips, mucosal surfaces, the face, and the chest. They are often associated with vascular disease (eg, pulmonary hypertension).

Any of these signs can be seen with full-blown systemic sclerosis, but CREST is usually much less aggressive and seldom leads to renal or congestive heart failure—the 2 leading causes of death in systemic sclerosis.

Diagnosis is based on recognition of the clinical signs and symptoms, as well as blood work (ie, antinuclear antibody and anti-centromere antibody tests). A skin biopsy is often performed to confirm the diagnosis.

Since there is no effective treatment for the overarching disease, the specific components of CREST are treated separately. For this patient, her lip lesions were treated with light electrodessication. Another option would have been laser treatment.

TAKE-HOME LEARNING POINTS

  • CREST syndrome, a limited form of systemic sclerosis, is a rare autoimmune condition caused by antibodies to cellular DNA and nuclear centromeres.
  • CREST is an acronym for the symptomatic manifestations of the condition: calcinosis, Raynaud disease, esophageal dysmotility, sclerodactyly, and telangiectasias.
  • Diagnosis is based on connecting the clinical dots and taking a thorough history, as well as lab results (for antinuclear antibodies and anti-centromere antibodies) and a skin biopsy.
  • CREST is usually treated symptomatically, one system at a time, since no definitive treatment exists for the disease itself.
Publications
Sections

A 39-year-old woman presents with red vascular streaks on her upper and lower lips. They have slowly multiplied and become more prominent since she first noticed them several years ago. Although the lesions are asymptomatic, their effect on her appearance bothers the patient.

Her primary care provider tried to resolve the problem with cryotherapy. But the treatment attempt was unsuccessful.

During history-taking, the patient divulges other existing health problems. She has been diagnosed with Raynaud syndrome, which flares several times a year, especially in cold weather or times of exceptional stress. She also has permanent thickening of the skin on her distal fingers, a result of sudden-onset swelling of all 10 fingers several years ago.

She denies any problems with eating, such as heartburn or difficulty swallowing. She also denies any respiratory problems or chronic fatigue.

Red vascular streaks on upper and lower lips

EXAMINATION
Seven very slender telangiectasias, most aligned vertically, are seen on the upper and lower vermillion surfaces. They range from a pinpoint to 6 mm in length. There are no similar lesions are on the rest of the oral mucosae, the face, or the chest.

The patient’s fingers, from the metacarpals to the tips, are decidedly edematous and firm but not tender. Several fingertips are scarred from past Raynaud episodes.

Swollen and scarred fingertips from Raynaud episodes

The patient looks her stated age and is in no distress.

What’s the diagnosis?

 

 

DISCUSSION
This patient almost certainly has CREST syndrome, a limited form of systemic sclerosis (or scleroderma). Both represent an autoimmune process in which antibodies attack cell DNA and centromeres (a component of the cell nucleus).

CREST can be difficult to diagnose because it can involve diverse organ systems. The name of the syndrome is an acronym for the symptoms it causes:

Calcinosis is a deposition of calcium triggered by inflammation. It manifests as small subcutaneous nodules, which are usually felt on the hands or seen on radiographs of the hands.

Raynaud syndrome causes intense vasoconstriction of blood vessels, usually in fingertips or toes, which first turn white, then red. The phenomenon is accompanied by pain and can be triggered by cold or stress.

Esophageal dysmotility occurs when atrophy and/or fibrosis of the esophageal lining leads to difficulty swallowing food.

Sclerodactyly is the term for tightening and thickening of the skin on the hands and fingers.

Telangiectasias are dilated capillaries that can worsen with time; they are common on lips, mucosal surfaces, the face, and the chest. They are often associated with vascular disease (eg, pulmonary hypertension).

Any of these signs can be seen with full-blown systemic sclerosis, but CREST is usually much less aggressive and seldom leads to renal or congestive heart failure—the 2 leading causes of death in systemic sclerosis.

Diagnosis is based on recognition of the clinical signs and symptoms, as well as blood work (ie, antinuclear antibody and anti-centromere antibody tests). A skin biopsy is often performed to confirm the diagnosis.

Since there is no effective treatment for the overarching disease, the specific components of CREST are treated separately. For this patient, her lip lesions were treated with light electrodessication. Another option would have been laser treatment.

TAKE-HOME LEARNING POINTS

  • CREST syndrome, a limited form of systemic sclerosis, is a rare autoimmune condition caused by antibodies to cellular DNA and nuclear centromeres.
  • CREST is an acronym for the symptomatic manifestations of the condition: calcinosis, Raynaud disease, esophageal dysmotility, sclerodactyly, and telangiectasias.
  • Diagnosis is based on connecting the clinical dots and taking a thorough history, as well as lab results (for antinuclear antibodies and anti-centromere antibodies) and a skin biopsy.
  • CREST is usually treated symptomatically, one system at a time, since no definitive treatment exists for the disease itself.

A 39-year-old woman presents with red vascular streaks on her upper and lower lips. They have slowly multiplied and become more prominent since she first noticed them several years ago. Although the lesions are asymptomatic, their effect on her appearance bothers the patient.

Her primary care provider tried to resolve the problem with cryotherapy. But the treatment attempt was unsuccessful.

During history-taking, the patient divulges other existing health problems. She has been diagnosed with Raynaud syndrome, which flares several times a year, especially in cold weather or times of exceptional stress. She also has permanent thickening of the skin on her distal fingers, a result of sudden-onset swelling of all 10 fingers several years ago.

She denies any problems with eating, such as heartburn or difficulty swallowing. She also denies any respiratory problems or chronic fatigue.

Red vascular streaks on upper and lower lips

EXAMINATION
Seven very slender telangiectasias, most aligned vertically, are seen on the upper and lower vermillion surfaces. They range from a pinpoint to 6 mm in length. There are no similar lesions are on the rest of the oral mucosae, the face, or the chest.

The patient’s fingers, from the metacarpals to the tips, are decidedly edematous and firm but not tender. Several fingertips are scarred from past Raynaud episodes.

Swollen and scarred fingertips from Raynaud episodes

The patient looks her stated age and is in no distress.

What’s the diagnosis?

 

 

DISCUSSION
This patient almost certainly has CREST syndrome, a limited form of systemic sclerosis (or scleroderma). Both represent an autoimmune process in which antibodies attack cell DNA and centromeres (a component of the cell nucleus).

CREST can be difficult to diagnose because it can involve diverse organ systems. The name of the syndrome is an acronym for the symptoms it causes:

Calcinosis is a deposition of calcium triggered by inflammation. It manifests as small subcutaneous nodules, which are usually felt on the hands or seen on radiographs of the hands.

Raynaud syndrome causes intense vasoconstriction of blood vessels, usually in fingertips or toes, which first turn white, then red. The phenomenon is accompanied by pain and can be triggered by cold or stress.

Esophageal dysmotility occurs when atrophy and/or fibrosis of the esophageal lining leads to difficulty swallowing food.

Sclerodactyly is the term for tightening and thickening of the skin on the hands and fingers.

Telangiectasias are dilated capillaries that can worsen with time; they are common on lips, mucosal surfaces, the face, and the chest. They are often associated with vascular disease (eg, pulmonary hypertension).

Any of these signs can be seen with full-blown systemic sclerosis, but CREST is usually much less aggressive and seldom leads to renal or congestive heart failure—the 2 leading causes of death in systemic sclerosis.

Diagnosis is based on recognition of the clinical signs and symptoms, as well as blood work (ie, antinuclear antibody and anti-centromere antibody tests). A skin biopsy is often performed to confirm the diagnosis.

Since there is no effective treatment for the overarching disease, the specific components of CREST are treated separately. For this patient, her lip lesions were treated with light electrodessication. Another option would have been laser treatment.

TAKE-HOME LEARNING POINTS

  • CREST syndrome, a limited form of systemic sclerosis, is a rare autoimmune condition caused by antibodies to cellular DNA and nuclear centromeres.
  • CREST is an acronym for the symptomatic manifestations of the condition: calcinosis, Raynaud disease, esophageal dysmotility, sclerodactyly, and telangiectasias.
  • Diagnosis is based on connecting the clinical dots and taking a thorough history, as well as lab results (for antinuclear antibodies and anti-centromere antibodies) and a skin biopsy.
  • CREST is usually treated symptomatically, one system at a time, since no definitive treatment exists for the disease itself.
Publications
Publications
Article Type
Display Headline
Blips on Lips
Display Headline
Blips on Lips
Sections
Disallow All Ads
Content Gating
No Gating (article Unlocked/Free)
Alternative CME
Disqus Comments
Default
Gate On Date
Tue, 09/24/2019 - 08:30
Un-Gate On Date
Tue, 09/24/2019 - 08:30
Use ProPublica
CFC Schedule Remove Status
Tue, 09/24/2019 - 08:30
Hide sidebar & use full width
render the right sidebar.

When Life’s an Itch

Article Type
Changed
Thu, 09/12/2019 - 00:01
Display Headline
When Life’s an Itch

A 34-year-old woman self-refers to dermatology for evaluation of a very itchy rash that manifested 2 weeks ago on her right arm. She immediately went to an urgent care clinic, where she was diagnosed with shingles and prescribed valacyclovir. This diagnosis was upsetting to the patient, as she was advised to avoid contact with her newborn niece for at least 2 weeks.

Despite the prescribed medication, however, the rash began to pop up in other areas, including her left arm, chest, and face. Through all of this, the patient felt fine: no fever, myalgia, or malaise.

Her husband suggested she seek an appointment with dermatology, which was expedited by a phone call from her primary care provider.

Poison ivy on arm

EXAMINATION
The patient is afebrile and in no acute distress. She is, however, quite upset with the widespread collections of vesicles on mildly erythematous bases, many in a linear configuration. In several areas, there is ecchymosis secondary to scratching.

What’s the diagnosis?

 

 

DISCUSSION
Poison ivy, or Rhus dermatitis, is one of the most common dermatologic problems seen in medicine—and yet, its various presentations can, as this case illustrates, be quite confusing. Even when it is recognized, treatment is far from satisfactory (but more on that later). Furthermore, there is a lot of misinformation about everything from the appearance of the offending plant to the condition’s “contagious” nature.

From a broader perspective, poison ivy is becoming more prevalent and its effects more pronounced as cities expand into formerly open country. The Rhus plant family (Toxicodendron radicans and others) thrives on our increasing levels of CO2, effectively making the “poisonous” resin in the stems, leaves, and berries more potent.

With repeated exposure, the vast majority of the population will develop an allergy to this resin, known as urushiol, which can persist even on long-dead plants, vines, and leaves. (It does take repeated exposure to develop the requisite T-cell population, which is why many children are immune to it.) The urushiol does not serve as a protective substance for the plant; rather, it helps the plant retain water. In fact, many animals feed on the plant with impunity.

Virtually all members of the poison ivy family display “leaves of three” emerging from a single stem, with each triplet alternating first on one side of the branch and then on the other. Several varieties of the plant flourish over vast areas of the world, but in the United States, east of the Rockies, Toxicodendron radicans is the dominant member of the family. It can grow as a low vine, a shrub, or a climbing vine, each with a distinct appearance aside from the leaves, which are almond-shaped, smooth, and usually shiny with smooth surfaces. Most mature leaves will have a single notch, sometimes called a “thumb,” on otherwise smooth, nonserrated edges. In the summer, tiny white and yellow berries begin to grow.

The climbing vines of older plants can reach heights of 10 meters or more. These vines can reach a thickness of 3 inches and often appear “furry,” with tiny rootlets covering their surfaces. Plants this large can produce leaves 12 to 14 inches long.

Clinically, the appearance of linear pink to red pruritic vesicular streaks typify this contact dermatitis, which can immediately follow exposure or take days to appear. Said exposure can be direct or via pets, tools, or aerosols (eg, from neighbors mowing their lawns). Besides avoidance of the great outdoors, washing thoroughly immediately after exposure makes sense (but many are unaware that they’ve been exposed until it’s too late).

Poison ivy is not contagious, though the general public firmly believes otherwise. Left untreated, it clears within 2 weeks (except in unusual cases). For those who cannot bear to wait, treatment is problematic, to say the least. OTC products, such as calamine lotion, do nothing for the itching but may help with blistering. Topical or systemic steroids reduce itching somewhat. Antihistamines are useless, since this condition does not involve histamine release.

TAKE-HOME LEARNING POINTS

  • Poison ivy (Rhus dermatitis) is quite common and becoming more so due to encroaching civilization and increasing CO2 levels.
  • “Leaves of three, let it be” is still good advice, because the poison ivy plant Toxicodendron radicans manifests with three almond-shaped, shiny, green leaves grouped in threes.
  • Urushiol is the name of the oily resin found in the plant’s stem, leaves, and berries, and is the trigger resulting in contact dermatitis.
  • Poison ivy is not contagious and typically clears in 2 weeks.
Publications
Topics
Sections

A 34-year-old woman self-refers to dermatology for evaluation of a very itchy rash that manifested 2 weeks ago on her right arm. She immediately went to an urgent care clinic, where she was diagnosed with shingles and prescribed valacyclovir. This diagnosis was upsetting to the patient, as she was advised to avoid contact with her newborn niece for at least 2 weeks.

Despite the prescribed medication, however, the rash began to pop up in other areas, including her left arm, chest, and face. Through all of this, the patient felt fine: no fever, myalgia, or malaise.

Her husband suggested she seek an appointment with dermatology, which was expedited by a phone call from her primary care provider.

Poison ivy on arm

EXAMINATION
The patient is afebrile and in no acute distress. She is, however, quite upset with the widespread collections of vesicles on mildly erythematous bases, many in a linear configuration. In several areas, there is ecchymosis secondary to scratching.

What’s the diagnosis?

 

 

DISCUSSION
Poison ivy, or Rhus dermatitis, is one of the most common dermatologic problems seen in medicine—and yet, its various presentations can, as this case illustrates, be quite confusing. Even when it is recognized, treatment is far from satisfactory (but more on that later). Furthermore, there is a lot of misinformation about everything from the appearance of the offending plant to the condition’s “contagious” nature.

From a broader perspective, poison ivy is becoming more prevalent and its effects more pronounced as cities expand into formerly open country. The Rhus plant family (Toxicodendron radicans and others) thrives on our increasing levels of CO2, effectively making the “poisonous” resin in the stems, leaves, and berries more potent.

With repeated exposure, the vast majority of the population will develop an allergy to this resin, known as urushiol, which can persist even on long-dead plants, vines, and leaves. (It does take repeated exposure to develop the requisite T-cell population, which is why many children are immune to it.) The urushiol does not serve as a protective substance for the plant; rather, it helps the plant retain water. In fact, many animals feed on the plant with impunity.

Virtually all members of the poison ivy family display “leaves of three” emerging from a single stem, with each triplet alternating first on one side of the branch and then on the other. Several varieties of the plant flourish over vast areas of the world, but in the United States, east of the Rockies, Toxicodendron radicans is the dominant member of the family. It can grow as a low vine, a shrub, or a climbing vine, each with a distinct appearance aside from the leaves, which are almond-shaped, smooth, and usually shiny with smooth surfaces. Most mature leaves will have a single notch, sometimes called a “thumb,” on otherwise smooth, nonserrated edges. In the summer, tiny white and yellow berries begin to grow.

The climbing vines of older plants can reach heights of 10 meters or more. These vines can reach a thickness of 3 inches and often appear “furry,” with tiny rootlets covering their surfaces. Plants this large can produce leaves 12 to 14 inches long.

Clinically, the appearance of linear pink to red pruritic vesicular streaks typify this contact dermatitis, which can immediately follow exposure or take days to appear. Said exposure can be direct or via pets, tools, or aerosols (eg, from neighbors mowing their lawns). Besides avoidance of the great outdoors, washing thoroughly immediately after exposure makes sense (but many are unaware that they’ve been exposed until it’s too late).

Poison ivy is not contagious, though the general public firmly believes otherwise. Left untreated, it clears within 2 weeks (except in unusual cases). For those who cannot bear to wait, treatment is problematic, to say the least. OTC products, such as calamine lotion, do nothing for the itching but may help with blistering. Topical or systemic steroids reduce itching somewhat. Antihistamines are useless, since this condition does not involve histamine release.

TAKE-HOME LEARNING POINTS

  • Poison ivy (Rhus dermatitis) is quite common and becoming more so due to encroaching civilization and increasing CO2 levels.
  • “Leaves of three, let it be” is still good advice, because the poison ivy plant Toxicodendron radicans manifests with three almond-shaped, shiny, green leaves grouped in threes.
  • Urushiol is the name of the oily resin found in the plant’s stem, leaves, and berries, and is the trigger resulting in contact dermatitis.
  • Poison ivy is not contagious and typically clears in 2 weeks.

A 34-year-old woman self-refers to dermatology for evaluation of a very itchy rash that manifested 2 weeks ago on her right arm. She immediately went to an urgent care clinic, where she was diagnosed with shingles and prescribed valacyclovir. This diagnosis was upsetting to the patient, as she was advised to avoid contact with her newborn niece for at least 2 weeks.

Despite the prescribed medication, however, the rash began to pop up in other areas, including her left arm, chest, and face. Through all of this, the patient felt fine: no fever, myalgia, or malaise.

Her husband suggested she seek an appointment with dermatology, which was expedited by a phone call from her primary care provider.

Poison ivy on arm

EXAMINATION
The patient is afebrile and in no acute distress. She is, however, quite upset with the widespread collections of vesicles on mildly erythematous bases, many in a linear configuration. In several areas, there is ecchymosis secondary to scratching.

What’s the diagnosis?

 

 

DISCUSSION
Poison ivy, or Rhus dermatitis, is one of the most common dermatologic problems seen in medicine—and yet, its various presentations can, as this case illustrates, be quite confusing. Even when it is recognized, treatment is far from satisfactory (but more on that later). Furthermore, there is a lot of misinformation about everything from the appearance of the offending plant to the condition’s “contagious” nature.

From a broader perspective, poison ivy is becoming more prevalent and its effects more pronounced as cities expand into formerly open country. The Rhus plant family (Toxicodendron radicans and others) thrives on our increasing levels of CO2, effectively making the “poisonous” resin in the stems, leaves, and berries more potent.

With repeated exposure, the vast majority of the population will develop an allergy to this resin, known as urushiol, which can persist even on long-dead plants, vines, and leaves. (It does take repeated exposure to develop the requisite T-cell population, which is why many children are immune to it.) The urushiol does not serve as a protective substance for the plant; rather, it helps the plant retain water. In fact, many animals feed on the plant with impunity.

Virtually all members of the poison ivy family display “leaves of three” emerging from a single stem, with each triplet alternating first on one side of the branch and then on the other. Several varieties of the plant flourish over vast areas of the world, but in the United States, east of the Rockies, Toxicodendron radicans is the dominant member of the family. It can grow as a low vine, a shrub, or a climbing vine, each with a distinct appearance aside from the leaves, which are almond-shaped, smooth, and usually shiny with smooth surfaces. Most mature leaves will have a single notch, sometimes called a “thumb,” on otherwise smooth, nonserrated edges. In the summer, tiny white and yellow berries begin to grow.

The climbing vines of older plants can reach heights of 10 meters or more. These vines can reach a thickness of 3 inches and often appear “furry,” with tiny rootlets covering their surfaces. Plants this large can produce leaves 12 to 14 inches long.

Clinically, the appearance of linear pink to red pruritic vesicular streaks typify this contact dermatitis, which can immediately follow exposure or take days to appear. Said exposure can be direct or via pets, tools, or aerosols (eg, from neighbors mowing their lawns). Besides avoidance of the great outdoors, washing thoroughly immediately after exposure makes sense (but many are unaware that they’ve been exposed until it’s too late).

Poison ivy is not contagious, though the general public firmly believes otherwise. Left untreated, it clears within 2 weeks (except in unusual cases). For those who cannot bear to wait, treatment is problematic, to say the least. OTC products, such as calamine lotion, do nothing for the itching but may help with blistering. Topical or systemic steroids reduce itching somewhat. Antihistamines are useless, since this condition does not involve histamine release.

TAKE-HOME LEARNING POINTS

  • Poison ivy (Rhus dermatitis) is quite common and becoming more so due to encroaching civilization and increasing CO2 levels.
  • “Leaves of three, let it be” is still good advice, because the poison ivy plant Toxicodendron radicans manifests with three almond-shaped, shiny, green leaves grouped in threes.
  • Urushiol is the name of the oily resin found in the plant’s stem, leaves, and berries, and is the trigger resulting in contact dermatitis.
  • Poison ivy is not contagious and typically clears in 2 weeks.
Publications
Publications
Topics
Article Type
Display Headline
When Life’s an Itch
Display Headline
When Life’s an Itch
Sections
Disallow All Ads
Content Gating
No Gating (article Unlocked/Free)
Alternative CME
Disqus Comments
Default
Gate On Date
Tue, 09/10/2019 - 10:30
Un-Gate On Date
Tue, 09/10/2019 - 10:30
Use ProPublica
CFC Schedule Remove Status
Tue, 09/10/2019 - 10:30
Hide sidebar & use full width
render the right sidebar.

An Atypical Problem for Atopical People

Article Type
Changed
Thu, 08/29/2019 - 00:01
Display Headline
An Atypical Problem for Atopical People

At age 1, a girl developed a blistery rash on the left side of her face. It was soon followed by a low-grade fever and modest malaise. All symptoms cleared within 2 weeks. Now, at age 4, she continues to experience similar, periodic outbreaks in the same location.

She has already been seen by various providers, including a dermatologist, and received several different diagnoses. The dermatologist scraped the rash and determined it to be a fungal infection. However, the recommended topical antifungal cream had no effect. At least 3 other providers (all nondermatology) called it cellulitis and treated with oral antibiotics, but these attempts also failed.

An Atypical Problem for Atopical People

EXAMINATION
There are no active lesions at the time of this initial examination and no palpable adenopathy in the region. There is a large area of erythema in a macular pattern over the right cheek. No scarring is visible.

The patient later returns when a new outbreak occurs. This time, there are distinct blisters and reactive adenopathy in the adjacent nodal areas.

What’s the diagnosis?

 

 

DISCUSSION
Results of a viral culture indicate herpes simplex.

The recurrence of persistent, vesicular rashes in the same location signifies a herpetic nature. Herpes simplex virus (HSV) is easier to diagnose in an adult patient, due to the ability to elicit a reliable history of premonitory symptoms. Small children have difficulty verbalizing the distinction between a tingle, an itch, and mild pain, which herald the onset of an HSV outbreak.

An episode of HSV can be triggered by anything that raises the body temperature (eg, stress, sickness, or sun exposure). Also important to note, these kinds of outbreaks can occur almost anywhere on the body, including ears, fingers, nipples, noses, and eyelids.

In my experience, most patients with longstanding herpes outbreaks are atopic (ie, allergy prone) or come from families in which atopy is common. Atopic patients are well known to be susceptible to all manner of skin infections, but most especially to herpes. It’s as if their immune systems overreact to pollen, mold, dust, and other allergens, while viral, fungal, and bacterial antigens fly under their immune radar.

In this case, the child was treated with valacyclovir on a chronic, as opposed to episodic, basis. With a bit of luck, this treatment will help to diminish HSV attacks as she matures.

TAKE-HOME LEARNING POINTS

  • Anything that raises body temperature (sun, colds, or even stress) can trigger an episode of herpes simplex virus (HSV).
  • Vesicular rashes that recur in the same location should be presumed herpetic, until proven otherwise. Usually, viral cultures aren’t necessary since the differential is so narrow.
  • Atopy can predispose one to all manner of skin infections, including viral, fungal, and bacterial.
  • Treatment of chronic HSV can be episodic or preventive, depending on the frequency and severity of attacks.
Publications
Topics
Sections

At age 1, a girl developed a blistery rash on the left side of her face. It was soon followed by a low-grade fever and modest malaise. All symptoms cleared within 2 weeks. Now, at age 4, she continues to experience similar, periodic outbreaks in the same location.

She has already been seen by various providers, including a dermatologist, and received several different diagnoses. The dermatologist scraped the rash and determined it to be a fungal infection. However, the recommended topical antifungal cream had no effect. At least 3 other providers (all nondermatology) called it cellulitis and treated with oral antibiotics, but these attempts also failed.

An Atypical Problem for Atopical People

EXAMINATION
There are no active lesions at the time of this initial examination and no palpable adenopathy in the region. There is a large area of erythema in a macular pattern over the right cheek. No scarring is visible.

The patient later returns when a new outbreak occurs. This time, there are distinct blisters and reactive adenopathy in the adjacent nodal areas.

What’s the diagnosis?

 

 

DISCUSSION
Results of a viral culture indicate herpes simplex.

The recurrence of persistent, vesicular rashes in the same location signifies a herpetic nature. Herpes simplex virus (HSV) is easier to diagnose in an adult patient, due to the ability to elicit a reliable history of premonitory symptoms. Small children have difficulty verbalizing the distinction between a tingle, an itch, and mild pain, which herald the onset of an HSV outbreak.

An episode of HSV can be triggered by anything that raises the body temperature (eg, stress, sickness, or sun exposure). Also important to note, these kinds of outbreaks can occur almost anywhere on the body, including ears, fingers, nipples, noses, and eyelids.

In my experience, most patients with longstanding herpes outbreaks are atopic (ie, allergy prone) or come from families in which atopy is common. Atopic patients are well known to be susceptible to all manner of skin infections, but most especially to herpes. It’s as if their immune systems overreact to pollen, mold, dust, and other allergens, while viral, fungal, and bacterial antigens fly under their immune radar.

In this case, the child was treated with valacyclovir on a chronic, as opposed to episodic, basis. With a bit of luck, this treatment will help to diminish HSV attacks as she matures.

TAKE-HOME LEARNING POINTS

  • Anything that raises body temperature (sun, colds, or even stress) can trigger an episode of herpes simplex virus (HSV).
  • Vesicular rashes that recur in the same location should be presumed herpetic, until proven otherwise. Usually, viral cultures aren’t necessary since the differential is so narrow.
  • Atopy can predispose one to all manner of skin infections, including viral, fungal, and bacterial.
  • Treatment of chronic HSV can be episodic or preventive, depending on the frequency and severity of attacks.

At age 1, a girl developed a blistery rash on the left side of her face. It was soon followed by a low-grade fever and modest malaise. All symptoms cleared within 2 weeks. Now, at age 4, she continues to experience similar, periodic outbreaks in the same location.

She has already been seen by various providers, including a dermatologist, and received several different diagnoses. The dermatologist scraped the rash and determined it to be a fungal infection. However, the recommended topical antifungal cream had no effect. At least 3 other providers (all nondermatology) called it cellulitis and treated with oral antibiotics, but these attempts also failed.

An Atypical Problem for Atopical People

EXAMINATION
There are no active lesions at the time of this initial examination and no palpable adenopathy in the region. There is a large area of erythema in a macular pattern over the right cheek. No scarring is visible.

The patient later returns when a new outbreak occurs. This time, there are distinct blisters and reactive adenopathy in the adjacent nodal areas.

What’s the diagnosis?

 

 

DISCUSSION
Results of a viral culture indicate herpes simplex.

The recurrence of persistent, vesicular rashes in the same location signifies a herpetic nature. Herpes simplex virus (HSV) is easier to diagnose in an adult patient, due to the ability to elicit a reliable history of premonitory symptoms. Small children have difficulty verbalizing the distinction between a tingle, an itch, and mild pain, which herald the onset of an HSV outbreak.

An episode of HSV can be triggered by anything that raises the body temperature (eg, stress, sickness, or sun exposure). Also important to note, these kinds of outbreaks can occur almost anywhere on the body, including ears, fingers, nipples, noses, and eyelids.

In my experience, most patients with longstanding herpes outbreaks are atopic (ie, allergy prone) or come from families in which atopy is common. Atopic patients are well known to be susceptible to all manner of skin infections, but most especially to herpes. It’s as if their immune systems overreact to pollen, mold, dust, and other allergens, while viral, fungal, and bacterial antigens fly under their immune radar.

In this case, the child was treated with valacyclovir on a chronic, as opposed to episodic, basis. With a bit of luck, this treatment will help to diminish HSV attacks as she matures.

TAKE-HOME LEARNING POINTS

  • Anything that raises body temperature (sun, colds, or even stress) can trigger an episode of herpes simplex virus (HSV).
  • Vesicular rashes that recur in the same location should be presumed herpetic, until proven otherwise. Usually, viral cultures aren’t necessary since the differential is so narrow.
  • Atopy can predispose one to all manner of skin infections, including viral, fungal, and bacterial.
  • Treatment of chronic HSV can be episodic or preventive, depending on the frequency and severity of attacks.
Publications
Publications
Topics
Article Type
Display Headline
An Atypical Problem for Atopical People
Display Headline
An Atypical Problem for Atopical People
Sections
Disallow All Ads
Content Gating
No Gating (article Unlocked/Free)
Alternative CME
Disqus Comments
Default
Gate On Date
Tue, 08/20/2019 - 10:45
Un-Gate On Date
Tue, 08/20/2019 - 10:45
Use ProPublica
CFC Schedule Remove Status
Tue, 08/20/2019 - 10:45
Hide sidebar & use full width
render the right sidebar.

Boiling Points

Article Type
Changed
Thu, 08/15/2019 - 00:01
Display Headline
Boiling Points

This 37-year-old woman began developing “boils” under both arms at age 12. Over the years, the lesions have become more numerous and bothersome. They are often painful and large and are capable of bursting on their own, releasing purulent material. Occasionally, similar lesions appear under her breasts and in the groin. The problem seems to wax and wane with her menstrual cycle. Family history reveals that both her mother and one of her sisters have had the same problem, again starting around the time of menarche.

Whenever the patient seeks medical care, usually at the emergency department, the diagnosis is always the same: boils. Normally, the prescribed treatment includes incision, drainage, and packing of the largest lesions, followed by 2 weeks of oral antibiotics. While the problem generally improves after treatment, it invariably returns.

Her health is decent overall. However, she has been overweight for years and has been smoking since she was 14.

Boiling Points

EXAMINATION
The patient’s left axilla shows ropy, hypertrophic scars, many comedones, and several fluctuant cystic subcutaneous masses. There is no frank erythema, although the patient indicates there often is.

No such changes are seen on examination of her right axilla. Instead, there is a slender 12-cm linear scar running across the axillary fold. Upon questioning, the patient reports that several years ago, a surgeon removed three-fourths of the skin and subcutaneous tissue from this area. This procedure cured the “boils” on her right arm, but it also left her with chronic lymphedema in that extremity.

Boiling Points

Other intertriginous areas are free of significant changes.

What’s the diagnosis?

 

 

DISCUSSION
In the US, hidradenitis suppurativa (HS) affects 1% to 4% of the population and about 4 times as many females as males. But as this case demonstrates, it is consistently misidentified as “boils” or “staph infection” by providers unfamiliar with the correct diagnosis.

HS involves hair follicles in intertriginous areas of the body that are rich with apocrine glands (eg, armpit, groin). The condition, initially known as acne inversa, was first described in 1833 by Dr. Alfred Velpeau, a French surgeon. Despite some minor similarities, HS is not actually a form of acne, nor is it an infection. About one-third of HS patients inherit the condition, and generally, onset occurs post puberty, suggesting a hormonal component.

With HS, the hair follicle and associated apocrine gland fail to function normally. As sweat accumulates in subcutaneous tissue, it creates a chronic inflammatory reaction manifesting with large comedones, cysts, and abscesses. Eventually, it can result in ropy, hypertrophic scars on the surface and deep tracts connecting multiple lesions. HS is classified as mild (stage 1), moderate (stage 2), or severe (stage 3) using the Hurley staging system.

HS is notoriously difficult to cure, but the anti-inflammatory effects of some antibiotics (eg, minocycline, doxycycline) can offer some relief, as can anti-androgens (eg, spironolactone). The use of isotretinoin has yielded disappointing results. For small lesions, intralesional injection of glucocorticoids can be useful for short-term relief of pain and swelling.

The most encouraging recent development in HS treatment is the approval for the use of adalimumab (Humira) in severe cases that have failed to respond to other modalities. Even with use of this biologic, decent control is probably the best outcome—and that’s at an annual cost of $50,000, plus the patient’s exposure to potentially serious adverse effects due to immunosuppression.

Another approach is surgical, with all its attendant risks, as this patient experienced in her right axilla. Simple incision and drainage offer little beyond temporary relief of pain.

Environmental factors should not be overlooked; obesity and smoking have both been linked to HS in multiple studies.

TAKE-HOME LEARNING POINTS

  • Hidradenitis suppurativa, also known as acne inversa, results from malfunction of the hair follicle and associated apocrine glands in intertriginous areas.
  • HS can range from mild (with minor pustules and sparse comedones) to and severe (diffuse disease, affecting multiple areas with heavy ropy scarring, large painful abscesses, and connecting tracts).
  • HS affects approximately 4 times as many females as males, almost all with post-pubertal onset—strongly suggestive of a hormonal component.
  • Treatment is problematic, although the recent approval of adalimumab for use in HS is proving to be helpful, if not curative. Some oral antibiotics and anti-androgens have shown mixed results.
Publications
Topics
Sections

This 37-year-old woman began developing “boils” under both arms at age 12. Over the years, the lesions have become more numerous and bothersome. They are often painful and large and are capable of bursting on their own, releasing purulent material. Occasionally, similar lesions appear under her breasts and in the groin. The problem seems to wax and wane with her menstrual cycle. Family history reveals that both her mother and one of her sisters have had the same problem, again starting around the time of menarche.

Whenever the patient seeks medical care, usually at the emergency department, the diagnosis is always the same: boils. Normally, the prescribed treatment includes incision, drainage, and packing of the largest lesions, followed by 2 weeks of oral antibiotics. While the problem generally improves after treatment, it invariably returns.

Her health is decent overall. However, she has been overweight for years and has been smoking since she was 14.

Boiling Points

EXAMINATION
The patient’s left axilla shows ropy, hypertrophic scars, many comedones, and several fluctuant cystic subcutaneous masses. There is no frank erythema, although the patient indicates there often is.

No such changes are seen on examination of her right axilla. Instead, there is a slender 12-cm linear scar running across the axillary fold. Upon questioning, the patient reports that several years ago, a surgeon removed three-fourths of the skin and subcutaneous tissue from this area. This procedure cured the “boils” on her right arm, but it also left her with chronic lymphedema in that extremity.

Boiling Points

Other intertriginous areas are free of significant changes.

What’s the diagnosis?

 

 

DISCUSSION
In the US, hidradenitis suppurativa (HS) affects 1% to 4% of the population and about 4 times as many females as males. But as this case demonstrates, it is consistently misidentified as “boils” or “staph infection” by providers unfamiliar with the correct diagnosis.

HS involves hair follicles in intertriginous areas of the body that are rich with apocrine glands (eg, armpit, groin). The condition, initially known as acne inversa, was first described in 1833 by Dr. Alfred Velpeau, a French surgeon. Despite some minor similarities, HS is not actually a form of acne, nor is it an infection. About one-third of HS patients inherit the condition, and generally, onset occurs post puberty, suggesting a hormonal component.

With HS, the hair follicle and associated apocrine gland fail to function normally. As sweat accumulates in subcutaneous tissue, it creates a chronic inflammatory reaction manifesting with large comedones, cysts, and abscesses. Eventually, it can result in ropy, hypertrophic scars on the surface and deep tracts connecting multiple lesions. HS is classified as mild (stage 1), moderate (stage 2), or severe (stage 3) using the Hurley staging system.

HS is notoriously difficult to cure, but the anti-inflammatory effects of some antibiotics (eg, minocycline, doxycycline) can offer some relief, as can anti-androgens (eg, spironolactone). The use of isotretinoin has yielded disappointing results. For small lesions, intralesional injection of glucocorticoids can be useful for short-term relief of pain and swelling.

The most encouraging recent development in HS treatment is the approval for the use of adalimumab (Humira) in severe cases that have failed to respond to other modalities. Even with use of this biologic, decent control is probably the best outcome—and that’s at an annual cost of $50,000, plus the patient’s exposure to potentially serious adverse effects due to immunosuppression.

Another approach is surgical, with all its attendant risks, as this patient experienced in her right axilla. Simple incision and drainage offer little beyond temporary relief of pain.

Environmental factors should not be overlooked; obesity and smoking have both been linked to HS in multiple studies.

TAKE-HOME LEARNING POINTS

  • Hidradenitis suppurativa, also known as acne inversa, results from malfunction of the hair follicle and associated apocrine glands in intertriginous areas.
  • HS can range from mild (with minor pustules and sparse comedones) to and severe (diffuse disease, affecting multiple areas with heavy ropy scarring, large painful abscesses, and connecting tracts).
  • HS affects approximately 4 times as many females as males, almost all with post-pubertal onset—strongly suggestive of a hormonal component.
  • Treatment is problematic, although the recent approval of adalimumab for use in HS is proving to be helpful, if not curative. Some oral antibiotics and anti-androgens have shown mixed results.

This 37-year-old woman began developing “boils” under both arms at age 12. Over the years, the lesions have become more numerous and bothersome. They are often painful and large and are capable of bursting on their own, releasing purulent material. Occasionally, similar lesions appear under her breasts and in the groin. The problem seems to wax and wane with her menstrual cycle. Family history reveals that both her mother and one of her sisters have had the same problem, again starting around the time of menarche.

Whenever the patient seeks medical care, usually at the emergency department, the diagnosis is always the same: boils. Normally, the prescribed treatment includes incision, drainage, and packing of the largest lesions, followed by 2 weeks of oral antibiotics. While the problem generally improves after treatment, it invariably returns.

Her health is decent overall. However, she has been overweight for years and has been smoking since she was 14.

Boiling Points

EXAMINATION
The patient’s left axilla shows ropy, hypertrophic scars, many comedones, and several fluctuant cystic subcutaneous masses. There is no frank erythema, although the patient indicates there often is.

No such changes are seen on examination of her right axilla. Instead, there is a slender 12-cm linear scar running across the axillary fold. Upon questioning, the patient reports that several years ago, a surgeon removed three-fourths of the skin and subcutaneous tissue from this area. This procedure cured the “boils” on her right arm, but it also left her with chronic lymphedema in that extremity.

Boiling Points

Other intertriginous areas are free of significant changes.

What’s the diagnosis?

 

 

DISCUSSION
In the US, hidradenitis suppurativa (HS) affects 1% to 4% of the population and about 4 times as many females as males. But as this case demonstrates, it is consistently misidentified as “boils” or “staph infection” by providers unfamiliar with the correct diagnosis.

HS involves hair follicles in intertriginous areas of the body that are rich with apocrine glands (eg, armpit, groin). The condition, initially known as acne inversa, was first described in 1833 by Dr. Alfred Velpeau, a French surgeon. Despite some minor similarities, HS is not actually a form of acne, nor is it an infection. About one-third of HS patients inherit the condition, and generally, onset occurs post puberty, suggesting a hormonal component.

With HS, the hair follicle and associated apocrine gland fail to function normally. As sweat accumulates in subcutaneous tissue, it creates a chronic inflammatory reaction manifesting with large comedones, cysts, and abscesses. Eventually, it can result in ropy, hypertrophic scars on the surface and deep tracts connecting multiple lesions. HS is classified as mild (stage 1), moderate (stage 2), or severe (stage 3) using the Hurley staging system.

HS is notoriously difficult to cure, but the anti-inflammatory effects of some antibiotics (eg, minocycline, doxycycline) can offer some relief, as can anti-androgens (eg, spironolactone). The use of isotretinoin has yielded disappointing results. For small lesions, intralesional injection of glucocorticoids can be useful for short-term relief of pain and swelling.

The most encouraging recent development in HS treatment is the approval for the use of adalimumab (Humira) in severe cases that have failed to respond to other modalities. Even with use of this biologic, decent control is probably the best outcome—and that’s at an annual cost of $50,000, plus the patient’s exposure to potentially serious adverse effects due to immunosuppression.

Another approach is surgical, with all its attendant risks, as this patient experienced in her right axilla. Simple incision and drainage offer little beyond temporary relief of pain.

Environmental factors should not be overlooked; obesity and smoking have both been linked to HS in multiple studies.

TAKE-HOME LEARNING POINTS

  • Hidradenitis suppurativa, also known as acne inversa, results from malfunction of the hair follicle and associated apocrine glands in intertriginous areas.
  • HS can range from mild (with minor pustules and sparse comedones) to and severe (diffuse disease, affecting multiple areas with heavy ropy scarring, large painful abscesses, and connecting tracts).
  • HS affects approximately 4 times as many females as males, almost all with post-pubertal onset—strongly suggestive of a hormonal component.
  • Treatment is problematic, although the recent approval of adalimumab for use in HS is proving to be helpful, if not curative. Some oral antibiotics and anti-androgens have shown mixed results.
Publications
Publications
Topics
Article Type
Display Headline
Boiling Points
Display Headline
Boiling Points
Sections
Disallow All Ads
Content Gating
No Gating (article Unlocked/Free)
Alternative CME
Disqus Comments
Default
Gate On Date
Tue, 08/06/2019 - 12:00
Un-Gate On Date
Tue, 08/06/2019 - 12:00
Use ProPublica
CFC Schedule Remove Status
Tue, 08/06/2019 - 12:00
Hide sidebar & use full width
render the right sidebar.

“Cupping” With Pain

Article Type
Changed
Thu, 08/01/2019 - 00:01
Display Headline
“Cupping” With Pain

A 30-year-old woman with a history of chronic overexposure to UV light presents to dermatology for a routine skin exam. The patient has a history of poor toleration to UV light, especially as a child, but participated in regular tanning as a teen. However, she stopped tanning when her sister developed a melanoma.

Additionally, the patient has been experiencing upper back pain, for which she has seen a variety of providers. Most recently, she consulted a naturopath, who recommended cupping therapy. Although the patient believes the therapy is alleviating her pain, she is distressed by the subsequent formation of large blemishes on her back and asks about possible treatment.

“Cupping” With Pain

EXAMINATION
There are 10 large round patches, each measuring 7 cm in diameter, on the patient’s back. These patches consist of multiple petechiae and brown hyperpigmentation. On palpation, there is no surface disturbance or tenderness. The discoloration is nonblanchable. The size, shape, and configuration of the lesions is consistent with the patient's description of the cupping procedures she has undergone on several occasions.

Notably, the patient's skin is categorized as type II on the Fitzpatrick scale, with advanced dermatoheliosis.

What’s the diagnosis?

 

 

DISCUSSION
"Cupping," as medical therapy, was first described in ancient texts 3000 to 4000 years ago. The application of cups to the patient’s skin was intended to draw out substances (eg, toxins and fluids) inside the body that were believed to cause a variety of ailments. Though its use has long since been discarded in mainstream medicine, it is still used routinely in both Chinese and alternative medicine.

Cupping has been evaluated by numerous medical individuals and organizations, who uniformly dismiss any benefit it might offer, even as a placebo. From a pathophysiologic standpoint, cupping causes localized dilation of blood and lymph vessels, thus creating telangiectasia that, as they resolve, leave behind postinflammatory hyperpigmentation and edema. (Excessive production of telangiectasia might indicate pathologic capillary fragility, possibly secondary to Rumpel-Leede phenomenon.)

The patient's skin type can affect the rate of resolution (longer for those with darker skin, shorter for those with fair skin); there is little we can do to speed up this process. Although the case patient was disappointed with the lack of available treatment for her blemishes, she was insistent about continuing the cupping therapy.

Interestingly, there is a differential diagnosis for such lesions; it includes injury from tennis balls, racquetballs, paintballs, or even baseballs—though the associated lesions are usually solitary.

TAKE-HOME LEARNING POINTS

  • Cupping, as medical therapy, has been around for thousands of years and is still routinely used in both Chinese and alternative medicine.
  • The intention of its use is to draw out noxious substances that purportedly cause the patient's complaint—however, according to numerous medical authorities, the practice is totally ineffective.
  • The suction effect of cupping induces edema and telangiectasia, which in turn results in postinflammatory hyperpigmentation that clears slowly.
  • Similar lesions can result from being struck by paintballs, racquetballs, tennis balls, and baseballs.
Publications
Topics
Sections

A 30-year-old woman with a history of chronic overexposure to UV light presents to dermatology for a routine skin exam. The patient has a history of poor toleration to UV light, especially as a child, but participated in regular tanning as a teen. However, she stopped tanning when her sister developed a melanoma.

Additionally, the patient has been experiencing upper back pain, for which she has seen a variety of providers. Most recently, she consulted a naturopath, who recommended cupping therapy. Although the patient believes the therapy is alleviating her pain, she is distressed by the subsequent formation of large blemishes on her back and asks about possible treatment.

“Cupping” With Pain

EXAMINATION
There are 10 large round patches, each measuring 7 cm in diameter, on the patient’s back. These patches consist of multiple petechiae and brown hyperpigmentation. On palpation, there is no surface disturbance or tenderness. The discoloration is nonblanchable. The size, shape, and configuration of the lesions is consistent with the patient's description of the cupping procedures she has undergone on several occasions.

Notably, the patient's skin is categorized as type II on the Fitzpatrick scale, with advanced dermatoheliosis.

What’s the diagnosis?

 

 

DISCUSSION
"Cupping," as medical therapy, was first described in ancient texts 3000 to 4000 years ago. The application of cups to the patient’s skin was intended to draw out substances (eg, toxins and fluids) inside the body that were believed to cause a variety of ailments. Though its use has long since been discarded in mainstream medicine, it is still used routinely in both Chinese and alternative medicine.

Cupping has been evaluated by numerous medical individuals and organizations, who uniformly dismiss any benefit it might offer, even as a placebo. From a pathophysiologic standpoint, cupping causes localized dilation of blood and lymph vessels, thus creating telangiectasia that, as they resolve, leave behind postinflammatory hyperpigmentation and edema. (Excessive production of telangiectasia might indicate pathologic capillary fragility, possibly secondary to Rumpel-Leede phenomenon.)

The patient's skin type can affect the rate of resolution (longer for those with darker skin, shorter for those with fair skin); there is little we can do to speed up this process. Although the case patient was disappointed with the lack of available treatment for her blemishes, she was insistent about continuing the cupping therapy.

Interestingly, there is a differential diagnosis for such lesions; it includes injury from tennis balls, racquetballs, paintballs, or even baseballs—though the associated lesions are usually solitary.

TAKE-HOME LEARNING POINTS

  • Cupping, as medical therapy, has been around for thousands of years and is still routinely used in both Chinese and alternative medicine.
  • The intention of its use is to draw out noxious substances that purportedly cause the patient's complaint—however, according to numerous medical authorities, the practice is totally ineffective.
  • The suction effect of cupping induces edema and telangiectasia, which in turn results in postinflammatory hyperpigmentation that clears slowly.
  • Similar lesions can result from being struck by paintballs, racquetballs, tennis balls, and baseballs.

A 30-year-old woman with a history of chronic overexposure to UV light presents to dermatology for a routine skin exam. The patient has a history of poor toleration to UV light, especially as a child, but participated in regular tanning as a teen. However, she stopped tanning when her sister developed a melanoma.

Additionally, the patient has been experiencing upper back pain, for which she has seen a variety of providers. Most recently, she consulted a naturopath, who recommended cupping therapy. Although the patient believes the therapy is alleviating her pain, she is distressed by the subsequent formation of large blemishes on her back and asks about possible treatment.

“Cupping” With Pain

EXAMINATION
There are 10 large round patches, each measuring 7 cm in diameter, on the patient’s back. These patches consist of multiple petechiae and brown hyperpigmentation. On palpation, there is no surface disturbance or tenderness. The discoloration is nonblanchable. The size, shape, and configuration of the lesions is consistent with the patient's description of the cupping procedures she has undergone on several occasions.

Notably, the patient's skin is categorized as type II on the Fitzpatrick scale, with advanced dermatoheliosis.

What’s the diagnosis?

 

 

DISCUSSION
"Cupping," as medical therapy, was first described in ancient texts 3000 to 4000 years ago. The application of cups to the patient’s skin was intended to draw out substances (eg, toxins and fluids) inside the body that were believed to cause a variety of ailments. Though its use has long since been discarded in mainstream medicine, it is still used routinely in both Chinese and alternative medicine.

Cupping has been evaluated by numerous medical individuals and organizations, who uniformly dismiss any benefit it might offer, even as a placebo. From a pathophysiologic standpoint, cupping causes localized dilation of blood and lymph vessels, thus creating telangiectasia that, as they resolve, leave behind postinflammatory hyperpigmentation and edema. (Excessive production of telangiectasia might indicate pathologic capillary fragility, possibly secondary to Rumpel-Leede phenomenon.)

The patient's skin type can affect the rate of resolution (longer for those with darker skin, shorter for those with fair skin); there is little we can do to speed up this process. Although the case patient was disappointed with the lack of available treatment for her blemishes, she was insistent about continuing the cupping therapy.

Interestingly, there is a differential diagnosis for such lesions; it includes injury from tennis balls, racquetballs, paintballs, or even baseballs—though the associated lesions are usually solitary.

TAKE-HOME LEARNING POINTS

  • Cupping, as medical therapy, has been around for thousands of years and is still routinely used in both Chinese and alternative medicine.
  • The intention of its use is to draw out noxious substances that purportedly cause the patient's complaint—however, according to numerous medical authorities, the practice is totally ineffective.
  • The suction effect of cupping induces edema and telangiectasia, which in turn results in postinflammatory hyperpigmentation that clears slowly.
  • Similar lesions can result from being struck by paintballs, racquetballs, tennis balls, and baseballs.
Publications
Publications
Topics
Article Type
Display Headline
“Cupping” With Pain
Display Headline
“Cupping” With Pain
Sections
Disallow All Ads
Content Gating
No Gating (article Unlocked/Free)
Alternative CME
Disqus Comments
Default
Gate On Date
Mon, 07/29/2019 - 11:30
Un-Gate On Date
Mon, 07/29/2019 - 11:30
Use ProPublica
CFC Schedule Remove Status
Mon, 07/29/2019 - 11:30
Hide sidebar & use full width
render the right sidebar.